Question and Answers Forum

All Questions      Topic List

Relation and Functions Questions

Previous in All Question      Next in All Question      

Previous in Relation and Functions      Next in Relation and Functions      

Question Number 15093 by Tinkutara last updated on 07/Jun/17

The range of f(x) = (({x}^2  − {x} + 1)/({x}^2  + {x} + 1));  (where {∙} denotes fractional function)  is?

$$\mathrm{The}\:\mathrm{range}\:\mathrm{of}\:{f}\left({x}\right)\:=\:\frac{\left\{{x}\right\}^{\mathrm{2}} \:−\:\left\{{x}\right\}\:+\:\mathrm{1}}{\left\{{x}\right\}^{\mathrm{2}} \:+\:\left\{{x}\right\}\:+\:\mathrm{1}}; \\ $$$$\left(\mathrm{where}\:\left\{\centerdot\right\}\:\mathrm{denotes}\:\mathrm{fractional}\:\mathrm{function}\right) \\ $$$$\mathrm{is}? \\ $$

Answered by mrW1 last updated on 07/Jun/17

 f(x) = (({x}^2  − {x} + 1)/({x}^2  + {x} + 1))  let g={x}  −1<g<1  f(x)=((g^2 −g+1)/(g^2 +g+1))=1−((2g)/(g^2 +g+1))  g→+1 f(x)→1−(2/3)=(1/3)  g=0, f(x)=1  g→−1 f(x)→3    ⇒f(x)∈((1/3),3)

$$\:{f}\left({x}\right)\:=\:\frac{\left\{{x}\right\}^{\mathrm{2}} \:−\:\left\{{x}\right\}\:+\:\mathrm{1}}{\left\{{x}\right\}^{\mathrm{2}} \:+\:\left\{{x}\right\}\:+\:\mathrm{1}} \\ $$$$\mathrm{let}\:\mathrm{g}=\left\{\mathrm{x}\right\} \\ $$$$−\mathrm{1}<\mathrm{g}<\mathrm{1} \\ $$$$\mathrm{f}\left(\mathrm{x}\right)=\frac{\mathrm{g}^{\mathrm{2}} −\mathrm{g}+\mathrm{1}}{\mathrm{g}^{\mathrm{2}} +\mathrm{g}+\mathrm{1}}=\mathrm{1}−\frac{\mathrm{2g}}{\mathrm{g}^{\mathrm{2}} +\mathrm{g}+\mathrm{1}} \\ $$$$\mathrm{g}\rightarrow+\mathrm{1}\:\mathrm{f}\left(\mathrm{x}\right)\rightarrow\mathrm{1}−\frac{\mathrm{2}}{\mathrm{3}}=\frac{\mathrm{1}}{\mathrm{3}} \\ $$$$\mathrm{g}=\mathrm{0},\:\mathrm{f}\left(\mathrm{x}\right)=\mathrm{1} \\ $$$$\mathrm{g}\rightarrow−\mathrm{1}\:\mathrm{f}\left(\mathrm{x}\right)\rightarrow\mathrm{3} \\ $$$$ \\ $$$$\Rightarrow\mathrm{f}\left(\mathrm{x}\right)\in\left(\frac{\mathrm{1}}{\mathrm{3}},\mathrm{3}\right) \\ $$

Commented by ajfour last updated on 07/Jun/17

0.3    i believe  [x] is the greatest integer that  the value of x has crossed going  along the numberline in the   positive x direction.    {x} is the value that need be  added to [x] to make it reach x .

$$\mathrm{0}.\mathrm{3}\:\:\:\:{i}\:{believe} \\ $$$$\left[{x}\right]\:{is}\:{the}\:{greatest}\:{integer}\:{that} \\ $$$${the}\:{value}\:{of}\:{x}\:{has}\:{crossed}\:{going} \\ $$$${along}\:{the}\:{numberline}\:{in}\:{the}\: \\ $$$${positive}\:{x}\:{direction}. \\ $$$$\:\:\left\{{x}\right\}\:{is}\:{the}\:{value}\:{that}\:{need}\:{be} \\ $$$${added}\:{to}\:\left[{x}\right]\:{to}\:{make}\:{it}\:{reach}\:\boldsymbol{{x}}\:. \\ $$

Commented by mrW1 last updated on 07/Jun/17

{−2.7}=−0.7

$$\left\{−\mathrm{2}.\mathrm{7}\right\}=−\mathrm{0}.\mathrm{7} \\ $$

Commented by mrW1 last updated on 07/Jun/17

(one) definition of fractional part of a number  see diagram:

$$\left(\mathrm{one}\right)\:\mathrm{definition}\:\mathrm{of}\:\mathrm{fractional}\:\mathrm{part}\:\mathrm{of}\:\mathrm{a}\:\mathrm{number} \\ $$$$\mathrm{see}\:\mathrm{diagram}: \\ $$

Commented by mrW1 last updated on 07/Jun/17

Commented by mrW1 last updated on 07/Jun/17

i have corrected. it is y∈((1/3),3)  you can check with x=−1.5:  {x}=−0.5  f(x)=(((−0.5)^2 −(−0.5)+1)/((−0.5)^2 +(−0.5)+1))=((1.75)/(0.75))=2.33>1

$$\mathrm{i}\:\mathrm{have}\:\mathrm{corrected}.\:\mathrm{it}\:\mathrm{is}\:\mathrm{y}\in\left(\frac{\mathrm{1}}{\mathrm{3}},\mathrm{3}\right) \\ $$$$\mathrm{you}\:\mathrm{can}\:\mathrm{check}\:\mathrm{with}\:\mathrm{x}=−\mathrm{1}.\mathrm{5}: \\ $$$$\left\{\mathrm{x}\right\}=−\mathrm{0}.\mathrm{5} \\ $$$$\mathrm{f}\left(\mathrm{x}\right)=\frac{\left(−\mathrm{0}.\mathrm{5}\right)^{\mathrm{2}} −\left(−\mathrm{0}.\mathrm{5}\right)+\mathrm{1}}{\left(−\mathrm{0}.\mathrm{5}\right)^{\mathrm{2}} +\left(−\mathrm{0}.\mathrm{5}\right)+\mathrm{1}}=\frac{\mathrm{1}.\mathrm{75}}{\mathrm{0}.\mathrm{75}}=\mathrm{2}.\mathrm{33}>\mathrm{1} \\ $$

Commented by Tinkutara last updated on 07/Jun/17

But answer is ((1/3), 1]

$$\mathrm{But}\:\mathrm{answer}\:\mathrm{is}\:\left(\frac{\mathrm{1}}{\mathrm{3}},\:\mathrm{1}\right] \\ $$

Commented by Tinkutara last updated on 07/Jun/17

Sir, if g = {x}, then 0 ≤ g < 1.  Why −1 < g < 1 ?

$$\mathrm{Sir},\:\mathrm{if}\:{g}\:=\:\left\{{x}\right\},\:\mathrm{then}\:\mathrm{0}\:\leqslant\:{g}\:<\:\mathrm{1}. \\ $$$$\mathrm{Why}\:−\mathrm{1}\:<\:{g}\:<\:\mathrm{1}\:? \\ $$

Commented by mrW1 last updated on 07/Jun/17

that is only for x≥0  for x≤0, −1<{x}≤0    {2.5}=0.5  {−2.5}=−0.5

$$\mathrm{that}\:\mathrm{is}\:\mathrm{only}\:\mathrm{for}\:\mathrm{x}\geqslant\mathrm{0} \\ $$$$\mathrm{for}\:\mathrm{x}\leqslant\mathrm{0},\:−\mathrm{1}<\left\{\mathrm{x}\right\}\leqslant\mathrm{0} \\ $$$$ \\ $$$$\left\{\mathrm{2}.\mathrm{5}\right\}=\mathrm{0}.\mathrm{5} \\ $$$$\left\{−\mathrm{2}.\mathrm{5}\right\}=−\mathrm{0}.\mathrm{5} \\ $$

Commented by Tinkutara last updated on 07/Jun/17

What will be {−2.7}?

$$\mathrm{What}\:\mathrm{will}\:\mathrm{be}\:\left\{−\mathrm{2}.\mathrm{7}\right\}? \\ $$

Commented by ajfour last updated on 07/Jun/17

0≤{x}<1 , i have read this.

$$\mathrm{0}\leqslant\left\{{x}\right\}<\mathrm{1}\:,\:{i}\:{have}\:{read}\:{this}. \\ $$

Commented by mrW1 last updated on 07/Jun/17

there are 2 kinds of definition for x<0  which are used in mathematic world.

$$\mathrm{there}\:\mathrm{are}\:\mathrm{2}\:\mathrm{kinds}\:\mathrm{of}\:\mathrm{definition}\:\mathrm{for}\:\mathrm{x}<\mathrm{0} \\ $$$$\mathrm{which}\:\mathrm{are}\:\mathrm{used}\:\mathrm{in}\:\mathrm{mathematic}\:\mathrm{world}. \\ $$

Commented by prakash jain last updated on 07/Jun/17

Fraction part in Indian textbooks  is defined as {x}=x−⌊x⌋  where ⌊∙⌋ is floor function.

$$\mathrm{Fraction}\:\mathrm{part}\:\mathrm{in}\:\mathrm{Indian}\:\mathrm{textbooks} \\ $$$$\mathrm{is}\:\mathrm{defined}\:\mathrm{as}\:\left\{{x}\right\}={x}−\lfloor{x}\rfloor \\ $$$$\mathrm{where}\:\lfloor\centerdot\rfloor\:\mathrm{is}\:\mathrm{floor}\:\mathrm{function}. \\ $$

Commented by Tinkutara last updated on 07/Jun/17

We have always  {x} = x − [x]  e.g. x ∈ Z (say 5), {5} = 5 − [5] = 0  x ∈ R^+  (say 4.8), {4.8} = 4.8 − [4.8]  = 4.8 − 4 = 0.8  x ∈ R^−  (say −3.78),  {−3.78} = −3.78 − [−3.78]  = −3.78 + 4 = 0.22  Hence {x} is always positive and  0 ≤ {x} < 1.  In this way {−0.5} = 0.5 and thus  f(−0.5) = (3/7)  (1/3) < f(−0.5) < 1

$$\mathrm{We}\:\mathrm{have}\:\mathrm{always} \\ $$$$\left\{{x}\right\}\:=\:{x}\:−\:\left[{x}\right] \\ $$$${e}.{g}.\:{x}\:\in\:{Z}\:\left(\mathrm{say}\:\mathrm{5}\right),\:\left\{\mathrm{5}\right\}\:=\:\mathrm{5}\:−\:\left[\mathrm{5}\right]\:=\:\mathrm{0} \\ $$$${x}\:\in\:{R}^{+} \:\left(\mathrm{say}\:\mathrm{4}.\mathrm{8}\right),\:\left\{\mathrm{4}.\mathrm{8}\right\}\:=\:\mathrm{4}.\mathrm{8}\:−\:\left[\mathrm{4}.\mathrm{8}\right] \\ $$$$=\:\mathrm{4}.\mathrm{8}\:−\:\mathrm{4}\:=\:\mathrm{0}.\mathrm{8} \\ $$$${x}\:\in\:{R}^{−} \:\left(\mathrm{say}\:−\mathrm{3}.\mathrm{78}\right), \\ $$$$\left\{−\mathrm{3}.\mathrm{78}\right\}\:=\:−\mathrm{3}.\mathrm{78}\:−\:\left[−\mathrm{3}.\mathrm{78}\right] \\ $$$$=\:−\mathrm{3}.\mathrm{78}\:+\:\mathrm{4}\:=\:\mathrm{0}.\mathrm{22} \\ $$$$\mathrm{Hence}\:\left\{{x}\right\}\:\mathrm{is}\:\mathrm{always}\:\mathrm{positive}\:\mathrm{and} \\ $$$$\mathrm{0}\:\leqslant\:\left\{{x}\right\}\:<\:\mathrm{1}. \\ $$$$\mathrm{In}\:\mathrm{this}\:\mathrm{way}\:\left\{−\mathrm{0}.\mathrm{5}\right\}\:=\:\mathrm{0}.\mathrm{5}\:\mathrm{and}\:\mathrm{thus} \\ $$$${f}\left(−\mathrm{0}.\mathrm{5}\right)\:=\:\frac{\mathrm{3}}{\mathrm{7}} \\ $$$$\frac{\mathrm{1}}{\mathrm{3}}\:<\:{f}\left(−\mathrm{0}.\mathrm{5}\right)\:<\:\mathrm{1} \\ $$

Commented by prakash jain last updated on 07/Jun/17

Wikipedia page on this has three  different definitions for fractional  part.  frac(x)=x−⌊x⌋  frac{x}= { ((x−⌊x⌋),(x≥0)),((x−⌈x⌉),(x<0)) :}  fract(x)=x−⌊∣x∣⌋  ⌊∙⌋=floor function  ⌈∙⌉=ceiling function  frac(−1.2) can take .8,−.2 or .2  depending upon the definition u use.  JEE exams will use {x}=x−⌊x⌋

$$\mathrm{Wikipedia}\:\mathrm{page}\:\mathrm{on}\:\mathrm{this}\:\mathrm{has}\:\mathrm{three} \\ $$$$\mathrm{different}\:\mathrm{definitions}\:\mathrm{for}\:\mathrm{fractional} \\ $$$$\mathrm{part}. \\ $$$${frac}\left({x}\right)={x}−\lfloor{x}\rfloor \\ $$$${frac}\left\{{x}\right\}=\begin{cases}{{x}−\lfloor{x}\rfloor}&{{x}\geqslant\mathrm{0}}\\{{x}−\lceil{x}\rceil}&{{x}<\mathrm{0}}\end{cases} \\ $$$${fract}\left({x}\right)={x}−\lfloor\mid{x}\mid\rfloor \\ $$$$\lfloor\centerdot\rfloor=\mathrm{floor}\:\mathrm{function} \\ $$$$\lceil\centerdot\rceil=\mathrm{ceiling}\:\mathrm{function} \\ $$$${frac}\left(−\mathrm{1}.\mathrm{2}\right)\:\mathrm{can}\:\mathrm{take}\:.\mathrm{8},−.\mathrm{2}\:\mathrm{or}\:.\mathrm{2} \\ $$$$\mathrm{depending}\:\mathrm{upon}\:\mathrm{the}\:\mathrm{definition}\:\mathrm{u}\:\mathrm{use}. \\ $$$$\mathrm{JEE}\:\mathrm{exams}\:\mathrm{will}\:\mathrm{use}\:\left\{{x}\right\}={x}−\lfloor{x}\rfloor \\ $$

Commented by mrW1 last updated on 07/Jun/17

thank you mr prakash jain for this  explanation.

$$\mathrm{thank}\:\mathrm{you}\:\mathrm{mr}\:\mathrm{prakash}\:\mathrm{jain}\:\mathrm{for}\:\mathrm{this} \\ $$$$\mathrm{explanation}. \\ $$

Answered by Tinkutara last updated on 08/Jun/17

Question done!  f(x) = 1 − ((2{x})/(({x} + (1/2))^2  + (3/4)))  Now assuming 0 ≤ {x} < 1,  we get 1 ≤ ({x} + (1/2))^2  + (3/4) < 3  ∴ When {x} = 0, f(x) = 1  And as {x} → 1, f(x) → 1 − (2/3) = (1/3)  ∴ f(x) is a decreasing function.  R_(f(x))  = ((1/3), 1]

$$\mathrm{Question}\:\mathrm{done}! \\ $$$${f}\left({x}\right)\:=\:\mathrm{1}\:−\:\frac{\mathrm{2}\left\{{x}\right\}}{\left(\left\{{x}\right\}\:+\:\frac{\mathrm{1}}{\mathrm{2}}\right)^{\mathrm{2}} \:+\:\frac{\mathrm{3}}{\mathrm{4}}} \\ $$$$\mathrm{Now}\:\mathrm{assuming}\:\mathrm{0}\:\leqslant\:\left\{{x}\right\}\:<\:\mathrm{1}, \\ $$$$\mathrm{we}\:\mathrm{get}\:\mathrm{1}\:\leqslant\:\left(\left\{{x}\right\}\:+\:\frac{\mathrm{1}}{\mathrm{2}}\right)^{\mathrm{2}} \:+\:\frac{\mathrm{3}}{\mathrm{4}}\:<\:\mathrm{3} \\ $$$$\therefore\:\mathrm{When}\:\left\{{x}\right\}\:=\:\mathrm{0},\:{f}\left({x}\right)\:=\:\mathrm{1} \\ $$$$\mathrm{And}\:\mathrm{as}\:\left\{{x}\right\}\:\rightarrow\:\mathrm{1},\:{f}\left({x}\right)\:\rightarrow\:\mathrm{1}\:−\:\frac{\mathrm{2}}{\mathrm{3}}\:=\:\frac{\mathrm{1}}{\mathrm{3}} \\ $$$$\therefore\:{f}\left({x}\right)\:\mathrm{is}\:\mathrm{a}\:\mathrm{decreasing}\:\mathrm{function}. \\ $$$$\mathrm{R}_{{f}\left({x}\right)} \:=\:\left(\frac{\mathrm{1}}{\mathrm{3}},\:\mathrm{1}\right] \\ $$

Terms of Service

Privacy Policy

Contact: info@tinkutara.com